Use app×
Join Bloom Tuition
One on One Online Tuition
JEE MAIN 2025 Foundation Course
NEET 2025 Foundation Course
CLASS 12 FOUNDATION COURSE
CLASS 10 FOUNDATION COURSE
CLASS 9 FOUNDATION COURSE
CLASS 8 FOUNDATION COURSE
+2 votes
148 views
in Statistics and probability by (575 points)
Let \( S=\left\{E_{1}, E_{2} \ldots . E_{8}\right\} \) be a sample space of random experiment such that \( P\left(E_{n}\right)=\frac{n}{36} \) for every \( n=1,2 \ldots 8 \). Then the number of elements in the set \( \left\{A \subset S: P(A) \geq \frac{4}{5}\right\} \) is

Please log in or register to answer this question.

1 Answer

0 votes
by (48.6k points)

If one of the number from {1, 2, ......, 8} is left then total \(a\ge29\) by 3 ways.

Similarly by leaving terms more 2 or 3 we get 16 more combinations.

\(\therefore\) Total number of different set A possible is 16 + 3 = 19

Related questions

Welcome to Sarthaks eConnect: A unique platform where students can interact with teachers/experts/students to get solutions to their queries. Students (upto class 10+2) preparing for All Government Exams, CBSE Board Exam, ICSE Board Exam, State Board Exam, JEE (Mains+Advance) and NEET can ask questions from any subject and get quick answers by subject teachers/ experts/mentors/students.

...